Bounded functions with unbounded integrals

Click For Summary
The discussion focuses on demonstrating the instability of the integrator by finding bounded functions that yield unbounded integrals. An initial example provided is the constant function, which leads to an output that increases indefinitely. The Heaviside step function is also mentioned, as its integral results in an unbounded output. Participants suggest that there are many functions, such as the arctangent function, which, while bounded, can still have an infinite area under the curve. The conversation emphasizes the challenge of identifying interesting examples beyond the basic ones already discussed.
roam
Messages
1,265
Reaction score
12

Homework Statement



I am trying to show that the integrator is unstable by giving examples of bounded inputs which produce unbounded outputs (i.e. a bounded function whose integral is unbounded).

Note: The integrator is a system which gives an output equal to the anti-derivative of its input.

The Attempt at a Solution



I have already proven the instability of the differentiator by considering the bounded input ##f(t)= \sin(t^2),## which gives the unbounded output ##f'(t) = 2t \cos(t^2).##

23h8yzp.jpg


For the integrator, I know, for instance, that the bounded input ##f(t)=1## gives the unbounded output ##t.## But could anyone suggest a more interesting example like the one I gave for the differentiator?

I couldn't come up with a good example. I would appreciate any suggestions or links.
 
Physics news on Phys.org
roam said:

Homework Statement



I am trying to show that the integrator is unstable by giving examples of bounded inputs which produce unbounded outputs (i.e. a bounded function whose integral is unbounded).

Note: The integrator is a system which gives an output equal to the anti-derivative of its input.

The Attempt at a Solution



I have already proven the instability of the differentiator by considering the bounded input ##f(t)= \sin(t^2),## which gives the unbounded output ##f'(t) = 2t \cos(t^2).##

View attachment 139989

For the integrator, I know, for instance, that the bounded input ##f(t)=1## gives the unbounded output ##t.## But could anyone suggest a more interesting example like the one I gave for the differentiator?

I couldn't come up with a good example. I would appreciate any suggestions or links.
Your problem might be more interesting on a bounded interval. Think about ##\sqrt[3] x## on ##(0,1)## for a differentiation example. But on a bounded interval no example exists for the integration because ##\left | \int_a^t f(u)~du\right | \le \int_a^b |f(u)|~du\le M(b-a)## where ##M## is a bound for ##|f|## on ##[a,b]##.
 
LCKurtz said:
Your problem might be more interesting on a bounded interval. Think about ##\sqrt[3] x## on ##(0,1)## for a differentiation example. But on a bounded interval no example exists for the integration because ##\left | \int_a^t f(u)~du\right | \le \int_a^b |f(u)|~du\le M(b-a)## where ##M## is a bound for ##|f|## on ##[a,b]##.

What about a non-bounded interval? I mean, I already found a function, ##f(t)=constant##, which increases indefinitely. The only other example I can think of, would be the Heaviside step function, ##f(t)=u(t)##. The output of the integrator for this function is:

$$g(t) = \intop^t_{-\infty} u(\tau) d\tau = \intop^t_{-\infty} 1 \ d\tau = t$$

for ##t>0.##

So, is there really no other function whose integral increases indefinitely? :confused:
 
roam said:
What about a non-bounded interval? I mean, I already found a function, ##f(t)=constant##, which increases indefinitely. The only other example I can think of, would be the Heaviside step function, ##f(t)=u(t)##. The output of the integrator for this function is:

$$g(t) = \intop^t_{-\infty} u(\tau) d\tau = \intop^t_{-\infty} 1 \ d\tau = t$$

for ##t>0.##

So, is there really no other function whose integral increases indefinitely? :confused:
Of course not. There are lots of examples. Take any function that is nonnegative for ##x\ge 0## that is bounded but has an infinite area. One such example is ##\arctan x,~0\le x## which is bounded by ##\frac \pi 2##.
 
  • Like
Likes roam
Question: A clock's minute hand has length 4 and its hour hand has length 3. What is the distance between the tips at the moment when it is increasing most rapidly?(Putnam Exam Question) Answer: Making assumption that both the hands moves at constant angular velocities, the answer is ## \sqrt{7} .## But don't you think this assumption is somewhat doubtful and wrong?

Similar threads

  • · Replies 16 ·
Replies
16
Views
3K
  • · Replies 3 ·
Replies
3
Views
2K
  • · Replies 8 ·
Replies
8
Views
2K
Replies
10
Views
2K
  • · Replies 5 ·
Replies
5
Views
2K
  • · Replies 23 ·
Replies
23
Views
2K
  • · Replies 1 ·
Replies
1
Views
2K
Replies
4
Views
2K
  • · Replies 3 ·
Replies
3
Views
2K
  • · Replies 1 ·
Replies
1
Views
2K